Skip to main content
Log in

On the dimension of Chowla–Milnor space

  • Published:
Proceedings - Mathematical Sciences Aims and scope Submit manuscript

Abstract

In a recent work, Gun, Murty and Rath defined the Chowla–Milnor space and proved a non-trivial lower bound for these spaces. They also obtained a conditional improvement of this lower bound and noted that an unconditional improvement of their lower bound will lead to irrationality of ζ(k)/ π k for odd positive integers k > 1. In this paper, we give an alternate proof of their theorem about the conditional lower bound.

This is a preview of subscription content, log in via an institution to check access.

Access this article

Price excludes VAT (USA)
Tax calculation will be finalised during checkout.

Instant access to the full article PDF.

Similar content being viewed by others

References

  1. Gun S, Ram Murty M and Rath P, On a conjecture of Chowla and Milnor, Canadian J. Math. 63(6) (2011) 1328–1344

    Article  MATH  Google Scholar 

  2. Okada T, On an extension of a theorem of S. Chowla, Acta Arith. 38(4) (1980/81) 341–345

    MathSciNet  Google Scholar 

Download references

Acknowledgement

The author would like to thank Sanoli Gun for helpful discussions.

Author information

Authors and Affiliations

Authors

Corresponding author

Correspondence to TAPAS CHATTERJEE.

Rights and permissions

Reprints and permissions

About this article

Cite this article

CHATTERJEE, T. On the dimension of Chowla–Milnor space. Proc Math Sci 122, 313–317 (2012). https://doi.org/10.1007/s12044-012-0088-1

Download citation

  • Received:

  • Revised:

  • Published:

  • Issue Date:

  • DOI: https://doi.org/10.1007/s12044-012-0088-1

Keywords

Navigation